www.vorkurse.de
Ein Projekt von vorhilfe.de
Die Online-Kurse der Vorhilfe

E-Learning leicht gemacht.
Hallo Gast!einloggen | registrieren ]
Startseite · Mitglieder · Teams · Forum · Wissen · Kurse · Impressum
Forenbaum
^ Forenbaum
Status Mathe-Vorkurse
  Status Organisatorisches
  Status Schule
    Status Wiederholung Algebra
    Status Einführung Analysis
    Status Einführung Analytisc
    Status VK 21: Mathematik 6.
    Status VK 37: Kurvendiskussionen
    Status VK Abivorbereitungen
  Status Universität
    Status Lerngruppe LinAlg
    Status VK 13 Analysis I FH
    Status Algebra 2006
    Status VK 22: Algebra 2007
    Status GruMiHH 06
    Status VK 58: Algebra 1
    Status VK 59: Lineare Algebra
    Status VK 60: Analysis
    Status Wahrscheinlichkeitst

Gezeigt werden alle Foren bis zur Tiefe 2

Navigation
 Startseite...
 Neuerdings beta neu
 Forum...
 vorwissen...
 vorkurse...
 Werkzeuge...
 Nachhilfevermittlung beta...
 Online-Spiele beta
 Suchen
 Verein...
 Impressum
Das Projekt
Server und Internetanbindung werden durch Spenden finanziert.
Organisiert wird das Projekt von unserem Koordinatorenteam.
Hunderte Mitglieder helfen ehrenamtlich in unseren moderierten Foren.
Anbieter der Seite ist der gemeinnützige Verein "Vorhilfe.de e.V.".
Partnerseiten
Weitere Fächer:

Open Source FunktionenplotterFunkyPlot: Kostenloser und quelloffener Funktionenplotter für Linux und andere Betriebssysteme
Forum "Determinanten" - Determinante
Determinante < Determinanten < Lineare Algebra < Hochschule < Mathe < Vorhilfe
Ansicht: [ geschachtelt ] | ^ Forum "Determinanten"  | ^^ Alle Foren  | ^ Forenbaum  | Materialien

Determinante: Beweis durch vollst. Induktion
Status: (Frage) überfällig Status 
Datum: 22:46 Mi 07.06.2006
Autor: oeli1985

Aufgabe
sei K ein Körper und a,b [mm] \in [/mm] K

A:=( [mm] a_{ij} [/mm] ) [mm] \in [/mm] M(nxn,K)
mit [mm] a_{ij}=\begin{cases} b, & \mbox{für } i=j \mbox{ } \\ a, & \mbox{ } sonst \mbox{ } \end{cases} [/mm]
Zeigen sie: det(A)= [mm] (b-a)^{n-1} [/mm] (b+ (n-1)a)

Hallo zusammen,

ich könnte etwas Hilfe bei dieser Aufgabe gebrauchen. Genauer gesagt brauche ich etwas Hilfe zum Induktionsschritt.

Ich habe selber 2 verschiedene Wege versucht, komme aber in beiden Fällen zum stocken.

Also:

Induktionsanfang

n=2: A=  [mm] \pmat{ b & a \\ a & b } [/mm] dann: det(A) = [mm] b^{2} [/mm] - [mm] a^{2} [/mm] = (b-a)(b+a)

Induktionsvoraussetzung

det(A)= [mm] (b-a)^{n-1} [/mm] (b+ (n-1)a)

Induktionsschritt

n [mm] \mapsto [/mm] n+1:

z.zg.: det(A)= [mm] (b-a)^{n} [/mm] (b+na) = [mm] (b-a)^{n-1} [/mm] ( [mm] b^{2} [/mm] + (n-1)ab - n [mm] a^{2} [/mm] )

dazu: det(A)= [mm] \summe_{i=1}^{n+1} (-1)^{n+1} a_{ij} [/mm] det [mm] A_{ij} [/mm] = ( [mm] \summe_{i=1}^{n} (-1)^{n+1} a_{ij} [/mm] det [mm] A_{ij} [/mm] ) [mm] \pm a_{n+1 1} [/mm] det [mm] A_{n+1 1} [/mm] = [mm] (b-a)^{n-1} [/mm] (b+(n-1)a) [mm] \pm [/mm] a det [mm] A_{n+1 1} [/mm] = [mm] (b-a)^{n-1} [/mm] (b+(n-1)a) [mm] \pm [/mm] a det B , wobei B die transponierte Matrix von [mm] A_{n+1 1} [/mm] ist.

Für B gilt: [mm] b_{ij}=\begin{cases} b, & \mbox{für } j=i+1 \mbox{ gerade} \\ a, & \mbox{ } sonst \mbox{ } \end{cases} [/mm]

[mm] (b-a)^{n-1} [/mm] (b+(n-1)a) [mm] \pm [/mm] a det B = [mm] (b-a)^{n-1} [/mm] (b+(n-1)a) [mm] \pm [/mm] a det C , wobei C durch Addition des (-1)-fachen der i-ten Zeile zur (i-1)-ten Zeile hervorgeht.

d.h.: Bei C handelt es sich um eine obere Dreiecksmatrix mit den Diagonaleinträgen [mm] c_{11} [/mm] = a und [mm] c_{ii} [/mm] = b-a für i=2,...,n

also:

[mm] (b-a)^{n-1} [/mm] (b+(n-1)a) [mm] \pm [/mm] a det C = [mm] (b-a)^{n-1} [/mm] (b+(n-1)a) [mm] \pm a^{2} (b-a)^{n-1} [/mm]

Ab diesem Punkt komme ich nicht weiter. Habe ich irgendwo einen Fehler gemacht oder finde ich einfach nur nicht die richtige Umformung?

Einen zweiten Ansatz zeige ich in der folgenden Mitteilung. Wäre nett, wenn ihr mir weiterhelfen könntet. DANKE

        
Bezug
Determinante: 2. Ansatz
Status: (Mitteilung) Reaktion unnötig Status 
Datum: 23:05 Mi 07.06.2006
Autor: oeli1985

Induktionsanfang: entspricht dem des 1. Ansatz
Induktionsvoraussetzung: entspricht der des 1. Ansatz
Induktionsschritt:

z.zg.: siehe 1. Ansatz
dazu:

det(a)= [mm] \summe_{i=1}^{n+1} (-1)^{(n+1)+j} a_{n+1 j} [/mm] det [mm] a_{n+1 j} [/mm] = [mm] (-1)^{n+2} a_{n+1 1} [/mm] det [mm] A_{n+1 1} [/mm] + [mm] (-1)^{n+3} a_{n+1 2} [/mm] det [mm] A_{n+1 2} [/mm] + ... + [mm] (-1)^{2n+2} a_{n+1 n+1} [/mm] det [mm] A_{n+1 n+1} [/mm] = [mm] a_{n+1 1} [/mm] det [mm] A_{n+1 1}- [/mm] ... + ... - ... + [mm] a_{n+1 n+1} [/mm] det [mm] A_{n+1 n+1} [/mm] = [mm] a_{n+1 1} [/mm] det [mm] A_{n+1 1}- [/mm] ... + ... - ... + b [mm] (b-a)^{n-1} [/mm] (b+(n-1)a)

d.h.:

es bleibt zu zeigen:

a [mm] \summe_{i=1}^{n} [/mm] det [mm] A_{n+1 i} [/mm] + b [mm] (b-a)^{n-1} [/mm] (b+(n-1)a) = [mm] (b-a)^{n} [/mm] (b+na)

Welcher Ansatz ist besser? Könnt ihr mir bei einem von beiden oder sogar beiden weiterhelfen? DANKE


Bezug
        
Bezug
Determinante: Antwort
Status: (Antwort) fertig Status 
Datum: 12:50 Sa 10.06.2006
Autor: felixf

Hallo!

> sei K ein Körper und a,b [mm]\in[/mm] K
>  
> A:=( [mm]a_{ij}[/mm] ) [mm]\in[/mm] M(nxn,K)
> mit [mm]a_{ij}=\begin{cases} b, & \mbox{für } i=j \mbox{ } \\ a, & \mbox{ } sonst \mbox{ } \end{cases}[/mm]
> Zeigen sie: det(A)= [mm](b-a)^{n-1}[/mm] (b+ (n-1)a)

Ich hab mir jetzt deine Ansaetze nicht durchgelesen, aber versuchs doch mal wie folgt mittels der Multilinearitaet der Determinante:

Die $n [mm] \times [/mm] n$-Matrix von dem Typ wie oben bezeichne mit [mm] $A_n$. [/mm] Dann ist
(fuer $n [mm] \ge [/mm] 3$) [mm] $\det A_n [/mm] = [mm] \det \pmat{ a & a & \cdots & a \\ a & & & \\ \vdots & & A_{n-1} & \\ a & & & } [/mm] + [mm] \det \pmat{ b-a & 0 & \cdots & 0 \\ a & & & \\ \vdots & & A_{n-1} & \\ a & & & }$. [/mm] Nun ist [mm] $\det \pmat{ b-a & 0 & \cdots & 0 \\ a & & & \\ \vdots & & A_{n-1} & \\ a & & & } [/mm] = (b - a) [mm] \det A_{n-1}$, [/mm] womit du hier Induktion anwenden kannst.

Nun zur anderen Determinante. Setze [mm] $B_n [/mm] := [mm] \pmat{ a & a & \cdots & a \\ a & & & \\ \vdots & & A_{n-1} & \\ a & & & }$. [/mm] Es ist (ebenfalls mit der Multilinearitaet) [mm] $\det B_n [/mm] = [mm] \det \pmat{ a & a & \cdots & a & a \\ a & & & & a \\ \vdots & & A_{n-2} & & \vdots \\ a & & & & a \\ a & a & \cdots & a & a } [/mm] + [mm] \det \pmat{ a & a & \cdots & a & a \\ a & & & & a \\ \vdots & & A_{n-2} & & \vdots \\ a & & & & a \\ 0 & 0 & \cdots & 0 & b - a }$. [/mm] Die erste Determinante ist gleich 0, da die erste mit der letzten Zeile der Matrix uebereinstimmt. Die Determinante der zweiten Matrix ist $(b - a) [mm] \det B_{n-1}$. [/mm] Hier musst du also ebenfalls Induktion verwenden (und erstmal selber eine Formel aufstellen; das ist jedoch recht einfach).

LG Felix


Bezug
        
Bezug
Determinante: Fälligkeit abgelaufen
Status: (Mitteilung) Reaktion unnötig Status 
Datum: 23:20 Sa 10.06.2006
Autor: matux

$MATUXTEXT(ueberfaellige_frage)
Bezug
Ansicht: [ geschachtelt ] | ^ Forum "Determinanten"  | ^^ Alle Foren  | ^ Forenbaum  | Materialien


^ Seitenanfang ^
www.vorkurse.de
[ Startseite | Mitglieder | Teams | Forum | Wissen | Kurse | Impressum ]